LSAT and Law School Admissions Forum

Get expert LSAT preparation and law school admissions advice from PowerScore Test Preparation.

 Administrator
PowerScore Staff
  • PowerScore Staff
  • Posts: 8927
  • Joined: Feb 02, 2011
|
#23566
Complete Question Explanation

Weaken. The correct answer choice is (B)

Quincy's reply basically boils down to — if it isn't broke, don't fix it. Since Quincy is supporting his argument by the fact that the system has always worked in the past, to weaken his argument we are likely going to be looking for something that has changed in the interim. The answer choice that addresses a sufficient change in the circumstances faced by current physicians will likely be the correct answer.

Answer Choice (A): This answer choice would clearly support Quincy's argument. If this system has worked in the past and nothing has changed, it follows that the system would work now.

Answer Choice (B): This is the correct answer choice. Quincy's argument is weakened because the circumstances faced by doctors today are more difficult than they were in the past. The doctors that Quincy is referring to may have been able to handle the long hours because the illnesses they were dealing with were easy to treat. The answer choice tells us that now the illnesses are more complex and therefore the opportunity for a tired doctor to make a mistake will be greater.

Answer Choice (C): Again, this answer choice supports Quincy's argument, as you would want the same physician to be working their shift during those first 24 hours to maintain continuity. If physicians were working shorter shifts, it would be more difficult to maintain continuity.

Answer Choice (D): This answer choice has no effect on the overall argument, as we have no information as to what specialties, if any, the differing viewpoints of Pamela and Quincy are referring to.

Answer Choice (E): Again, this answer choice does nothing to address the primary question of whether or not physicians should or should not be working 36 hour shifts.
 Jack.darb
  • Posts: 2
  • Joined: Aug 19, 2016
|
#27939
I was doing 21-45 timed and got this one wrong. Going over the answer choices again, I had to refrain from slapping myself for getting this one wrong. The question asks for the best response, and I noticed that B "because ..." Is the only one that's fit for a proper response to Quincys question. Do the test makers do this often? The answer to this type of question being the only choice that has "because..."
 Adam Tyson
PowerScore Staff
  • PowerScore Staff
  • Posts: 5153
  • Joined: Apr 14, 2011
|
#27940
Hey Jack, don't slap yourself! Mistakes happen, and we can learn from them.

I would advise against getting too fixated on the language used. That word "because" could as easily have introduced a wrong answer. Answer A, for example, could have been "because the basic responsibilities haven't changed". Instead, focus on the prephrase - here, you should be looking for an answer that tells us that something has changed that makes the old way less effective that it was. Answer B does that by changing the circumstances under which these doctors are now working.

I would say this is actually a bit unusual, not common, in that the authors like to give wrong answers that use language very similar to the right answer, or even give the wrong answer attractive language and make the wrong answer kind of ugly. Here, the right answer has some very helpful language in that "because" that makes it stand out from the wrong answers. This was very friendly of the authors, in my opinion, and I would not count on their being that friendly very often.

Focus on the prephrase, and on the overall meaning of the stimulus, stem, and answer choices, and less on particular words or word combinations that may seem enticing. Those will often be used to lay traps for the unsuspecting test taker. In the words of Admiral Akbar, "It's a trap!"
 mrcheese
  • Posts: 32
  • Joined: Jun 27, 2018
|
#57330
I picked D.

Quincy said that physicians "generally" make good decisions.

"generally" - in most cases; usually.

What if the specialties that are the exceptions to the "generally made good decisions portion" make plenty of bad decisions, because their specialty is more difficult, such as surgery? I assumed when it says "physicians" it means all physicians.

I can see why B is better, but only because it addresses, "Why should what has worked in the past be changed now?"

However, I would think that the response to B by Quincy would just be, "Well, yeah. Patients are more seriously ill now than they were in the past, but they [the physicians] still generally make good decisions."

Quincy remains unconvinced. No problem, because there is no challenge.

If some specialties have things more difficult, then they could be the reason physicians only "generally" make good decisions. A few mistakes by a few people.

Maybe I am adding information?
 Claire Horan
PowerScore Staff
  • PowerScore Staff
  • Posts: 408
  • Joined: Apr 18, 2016
|
#60877
Hi Mr. Cheese,

I think there are two problems with your analysis. First, there is nothing in the stimulus that suggests that the specialty physicians who were trained working long hours are not part of the group that generally made good decisions. I think you are right that you are making an assumption there that is not warranted by the stimulus.

Secondly, answer choice D doesn't focus on the discrepancy in the question. That's why prephrasing is so handy. If you prephrased this questions, chances are your answer would focus on the question of what has changed, since the long hours seem to have not been a problem in the past. By prephrasing, you avoid getting distracted by an answer choice.

Along those lines, try not to persuade yourself that answer choice fits by thinking thoughts like, "This answer choice could work because maybe ____ or if _____." The right answer should be apparent from the information given, without your needing to supply additional assumptions.

Good luck!
 Leela
  • Posts: 63
  • Joined: Apr 13, 2019
|
#64300
When crossing off answers as contenders, at what point do you advise getting rid of an answer because it discusses information we don't care about? I read "medical reimbursement policies" in B and got rid of it because I didn't see a direct connection to how it the work of physicians. I ended up choosing D.
User avatar
 Dave Killoran
PowerScore Staff
  • PowerScore Staff
  • Posts: 5848
  • Joined: Mar 25, 2011
|
#64301
Leela wrote:When crossing off answers as contenders, at what point do you advise getting rid of an answer because it discusses information we don't care about? I read "medical reimbursement policies" in B and got rid of it because I didn't see a direct connection to how it the work of physicians. I ended up choosing D.
Hi Leela,

Good question! the answer is that ti depends on the question type. In questions like Must or Flaw, we are engaging in a strictly fact-based exercise where irrelevant items are typically distractors. But in questions such as Strengthen or Weaken (such as this one), the scope of what is relevant widens significantly. Outside information is acceptable, and you have to be very careful about eliminating answers that initially look irrelevant because they can come up with ways to add relevant information to them, or they could open up an angle you hadn't previously considered (or that hadn't been discussed).

So, in (B), while it initially seems like the info isn't relevant, note hat happens after the first few words: they raise an issue that is relevant to Quincy's argument. The first few words may be on a topic that didn't previously come up, but it is relevant to patients and ultimately gives us info that would help us counter Quincy.

The beauty of this is that you've encountered this before the LSAT :-D Now you know this is what they do at times (and this is intentional on their part without question), so next time you'll be more likely to read past that seemingly irrelevant info to see if there's something useful afterwards.

Thanks!
 Mozart
  • Posts: 7
  • Joined: Apr 16, 2020
|
#75945
Hi PowerScore,

In addition to not addressing the primary flaw in the stimulus, does E) also strengthen Quincy's argument? This answer choice seems to suggest that physicians should work at least 36 hours to observe and recognize the patients.

I really appreciate your help!
User avatar
 KelseyWoods
PowerScore Staff
  • PowerScore Staff
  • Posts: 1079
  • Joined: Jun 26, 2013
|
#76050
Hi Mozart!

No, answer choice (E) would not strengthen Quincy's argument. Quincy's argument is that since the doctors followed this regimen during their training period and made good decisions, they should be able to still make good decisions while working long hours. Answer choice (E) is about what training should include, but what training should include really has nothing to do with whether or not doctors who were already trained one way should be able to continue working that way.

Hope this helps!

Best,
Kelsey
User avatar
 sdb606
  • Posts: 78
  • Joined: Feb 22, 2021
|
#85773
My reason for picking E was Pamela could say, "The data could show us if patients tend to decline at the end of a resident's shift. This would support my argument that fatigue impairs ability at the end of a shift and counter Quincy's argument by showing that things need to be changed." My unstated assumption here is that patients declining is a bad thing that should be prevented with a change in physician training.

I eliminated B because even though patients are more seriously ill on average, physicians still generally make good decisions, according to Quincy, so it doesn't matter if they are more seriously ill. To state that physicians are NOT making good decisions would be challenging a premise which I thought was not to be done in a Weaken question.

Get the most out of your LSAT Prep Plus subscription.

Analyze and track your performance with our Testing and Analytics Package.